Inverse Laplace involving heaviside function

Click For Summary
SUMMARY

The inverse Laplace transform of the function F(s) = (-3/s) + (e^-4s)/(s^2) + (3e^-4s)/s is calculated in three parts. The first part yields -3, the second part results in u(x-4)x, and the third part gives 3u(x-4). The correct final expression is f(x) = u(x-4)x - 3 - u(x-4), which simplifies to -3 + u(x-4)(x - 4). The confusion arose from the interpretation of the Heaviside function and its impact on the final expression.

PREREQUISITES
  • Understanding of inverse Laplace transforms
  • Familiarity with Heaviside step functions
  • Knowledge of Laplace transform properties
  • Basic algebraic manipulation skills
NEXT STEPS
  • Study the properties of the Heaviside function in Laplace transforms
  • Learn about the convolution theorem in Laplace transforms
  • Explore examples of inverse Laplace transforms involving exponential shifts
  • Practice solving inverse Laplace problems using MATLAB or Mathematica
USEFUL FOR

Students studying differential equations, engineers applying Laplace transforms in control systems, and mathematicians focusing on transform techniques.

reed2100
Messages
49
Reaction score
1

Homework Statement


Give the inverse Laplace transform of F(s) = (-3/s) + (e^-4s)/(s^2) + (3e^-4s)/s

Homework Equations


Inverse Laplace [e^(-cs) F(s)] = f(x-c)u(x-c)

The Attempt at a Solution



I'll break this into 3 parts.

Part 1 - (-3/s)
-3/s = -3(1/s) -> inverse laplace of -3(1/s) = -3

Part 2 - (e^(-4s))/(s^2)
By looking at the e component, I see that in this case "c" = 4.

Rearrange to (e^(-4s)) (1/s^2)
So the inverse laplace = u(x-4)*x [ u(x-4) being for the heaviside component and x being the inverse laplace of 1/s^2]

Part 3 - (3e^-4s / s)

e component still says that c = 4.
Rearrange into 3(e^-4s) (1/s)
Inverse laplace becomes 3 * u(x-4) * 1 = 3u(x-4)

So MY answer would be that the inverse laplace of the complete F(s) is

-3 + u(x-4)x + 3u(x-4)

The CORRECT answer that wolfram spat out and that is an answer choice on the online practice test is
f(x) = u(x-4)x - 3 - u(x-4)

I see the similarities but that makes it even more confusing to me how they got
-u(x-4) instead of +3u(x-4). I don't see how that works out.
Any and all help is greatly appreciated.
 
Physics news on Phys.org
reed2100 said:

Homework Statement


Give the inverse Laplace transform of F(s) = (-3/s) + (e^-4s)/(s^2) + (3e^-4s)/s

Homework Equations


Inverse Laplace [e^(-cs) F(s)] = f(x-c)u(x-c)

The Attempt at a Solution



I'll break this into 3 parts.

Part 1 - (-3/s)
-3/s = -3(1/s) -> inverse laplace of -3(1/s) = -3

Part 2 - (e^(-4s))/(s^2)
By looking at the e component, I see that in this case "c" = 4.

Rearrange to (e^(-4s)) (1/s^2)
So the inverse laplace = u(x-4)*x [ u(x-4) being for the heaviside component and x being the inverse laplace of 1/s^2]

Part 3 - (3e^-4s / s)

e component still says that c = 4.
Rearrange into 3(e^-4s) (1/s)
Inverse laplace becomes 3 * u(x-4) * 1 = 3u(x-4)

So MY answer would be that the inverse laplace of the complete F(s) is

-3 + u(x-4)x + 3u(x-4)

The CORRECT answer that wolfram spat out and that is an answer choice on the online practice test is
f(x) = u(x-4)x - 3 - u(x-4)

I see the similarities but that makes it even more confusing to me how they got
-u(x-4) instead of +3u(x-4). I don't see how that works out.
Any and all help is greatly appreciated.

You should have -3 + u(x-4)(x-4) + 3u(x-4)
 
Ray Vickson said:
You should have -3 + u(x-4)(x-4) + 3u(x-4)
Oh ok, I see it now. That's a careless mistake. So when fixing that , the u(x-4)(x-4) becomes u(x-4)x - u(x-4)4.
Combining it all together you get -3 + u(x-4)x -1u(x-4).

Thank you for your help!
 
Question: A clock's minute hand has length 4 and its hour hand has length 3. What is the distance between the tips at the moment when it is increasing most rapidly?(Putnam Exam Question) Answer: Making assumption that both the hands moves at constant angular velocities, the answer is ## \sqrt{7} .## But don't you think this assumption is somewhat doubtful and wrong?

Similar threads

  • · Replies 10 ·
Replies
10
Views
2K
  • · Replies 1 ·
Replies
1
Views
1K
Replies
2
Views
2K
  • · Replies 8 ·
Replies
8
Views
2K
Replies
2
Views
2K
  • · Replies 9 ·
Replies
9
Views
3K
  • · Replies 2 ·
Replies
2
Views
1K
Replies
3
Views
2K
Replies
9
Views
2K
  • · Replies 8 ·
Replies
8
Views
3K